LMFT Missed questions

Lakukan tugas rumah & ujian kamu dengan baik sekarang menggunakan Quizwiz!

A Chinese-speaking therapist moves to a new city with a relatively large Chinese American population. The therapist would like to market her mental health services to the Chinese population. What factor tends to be the largest barrier to Asian-Americans utilizing mental health services? Response: A. Lack of insurance B. Language issues C. Shame and stigma D. Reliance on traditional medicine

C. Shame and stigma

A therapist has been meeting with an El Salvadorian family for eight months on a sliding scale fee. The family informs the therapist they plan to bring homemade food to the final session as a thank you for all that the therapist has done. How should the therapist respond? Response: A. Accept the offer and thank the clients for their thoughtfulness. B. Deny the offer and explain the therapist cannot accept gifts. C. Seek consultation to determine the appropriateness of the gesture. D. Determine if giving a gift is part of the El Salvadorian cultural norm before accepting or declining the gift.

A. Accept the offer and thank the clients for their thoughtfulness.

Sally, a 66-year-old widow who lost her husband five months ago, is self-referred. Sally recently retired from her job at the post office. Lately, she has lost her appetite and has been having trouble sleeping and concentrating. She reports that the only thing she enjoys anymore is spending time with her 8-year-old granddaughter. She takes her to school and to ballet practice. Otherwise, Sally just feels useless and is concerned that she will become a burden to her daughter's family. She tears up when she says, "I'm having a hard time getting old. I hate being alone. I still pretend my husband is just at work. I keep praying that he'll come back. I now know why my husband used to refer to his drink as 'old faithful.'" What initial interventions should the therapist consider in the case provided in the vignette? Response: A. Acknowledge the changes that have occurred in her life and assess her safety. B. Identify potential volunteer opportunities and assess her level of alcohol use. C. Provide psychoeducation about life stages and assess her safety. D. Determine her level of social support and provide psychoeducation about grief and loss.

A. Acknowledge the changes that have occurred in her life and assess her safety. Rationale: Answer (A) is the best answer. You'd want to start by acknowledging what she's been through and finding out if she has any thoughts of harming herself. (B) is jumping too quickly to having her volunteer. Psychoeducation about life stages could be done, but acknowledging the changes would come first (C). Determining her level of social support is more of an assessment activity than an intervention (D).

A 34-year-old single man seeks therapy for conflict at work and with his family. During the assessment, he shares that he frequently explodes at people and "he isn't sure where the anger is coming from." He tells the therapist that he's gotten a verbal warning at work and he and his older brother haven't spoken in weeks because of an outburst he had. After completing the assessment and identifying the treatment goals, how would a Psychodynamic therapist begin the initial therapy session? Response: A. Explore how the client's week was. B. Wait silently and attentively for the client to speak. C. Remind the client of his treatment goals and ask what he'd like to address first. D. Inquire about the client's early childhood relationship.

B. Wait silently and attentively for the client to speak. Rationale: The question is asking about the way in which a Psychodynamic therapist would begin a session. Psychodynamic therapists are neutral and non-directive, so they will not initiate the conversation but wait for the client to start. Thus (B) is the best answer. Answers (A) and (C) makes the therapist too direct and involved for it to be a Psychodynamic therapist. Answer (D) might happen during the later stages, but the therapist is not going to start with such intervention.

Michael, a 25 year-old African-American man, is court-ordered to therapy after being released from prison, where he spent time for selling drugs. Michael states that he is only there because he is required to come. Under his breath he says, "The whole system is rigged against the black man." How should the therapist respond to the client in this case? Response: A. "Tell me what you mean by 'This whole system is rigged against the black man.'" B. "How have you been adjusting to life outside of prison?" C. "I imagine you've experienced a lot of racism." D. "It is up to you how much you get out of therapy."

C. "I imagine you've experienced a lot of racism."

A therapist admits to an intimate relationship with one of his clients whom he has treated for over two years. The therapist is the subject of regulatory board investigation. He tells the investigator that he truly loves this client and would like to marry her. When interviewed, the client reports that she initiated the sexual contact with the therapist. The therapist terminated therapy with the client when she moved in with him. The therapist's actions are: Response: A. a violation of boundary. B. legal but unethical. C. acceptable because the therapeutic relationship was terminated. D. illegal and unethical.

D. illegal and unethical. Rationale: The actions described in this stem are both illegal and unethical (D). The actions are violations of boundary, but the ethical and legal implications are more important and better response than answer (A). Answer (B) is inaccurate as the actions are illegal as well and answer (C) is an inaccurate statement—based on the information provided in the stem the therapeutic relationship overlapped with the intimate relationship.

A 36-year-old woman seeks therapy for depression following a recent break up with her long-term boyfriend. During the initial session, the client begins sobbing, has a difficult time breathing and is unable to speak. How would a Gestalt therapist proceed? Response: A. Use an empty chair to help the client process her feelings about the boyfriend. B. Acknowledge her difficulty speaking and suggest the client to take deep breaths to help her calm down. C. Sit quietly until the client calms down. D. Ask the client if there is anything the therapist could do to support her.

B. Acknowledge her difficulty speaking and suggest the client to take deep breaths to help her calm down.

A family is referred by their child's therapist for family therapy. The mother reports that she is basically a "single parent" because her husband works so much. The two young children sit with the mom on the couch and the husband sits by himself in the corner chair. How would a Structural family therapist proceed in this case? Response: A. Create a family map to explore the roles of the parents' family of origin. B. Align with the father. C. Explore the parent's beliefs about parenting roles. D. Direct the father to work more.

B. Align with the father.

Molly, a 23 year-old college student, comes to treatment complaining of constant and frequent mood swings. During the assessment phase, the therapist learns that Molly has been diagnosed with diabetes but has not been getting proper treatment for it. She exclaims: "If I monitor what I eat and maintain a proper diet, I am OK. I do not have health insurance and it is too expensive to pay for doctor's visits out of pocket. Could you just help me figure out the number of sugar calories that I consume?" What would the treatment goals be during the initial stages of treatment for a Narrative therapist? Response: A. Separate the client from the problem; explore the story she is telling herself around her illness. B. Explain that you are not trained to help her with managing her diabetes; assist the client in finding medical resources. C. Provide the client with specific tools that can help Molly monitor her diet; record progress of the client throughout treatment. D. Identify factors that enhance the problem; help client reduce harm inducing behaviors.

B. Explain that you are not trained to help her with managing her diabetes; assist the client in finding medical resources. Rationale: The symptoms being described and the goals of the client are outside the scope of practice for a therapist. The client requires medical services and the therapist's initial goal would be to connect the client with those services (B). Although externalizing the problem is part of Narrative Therapy (A), it is not an appropriate intervention, because it does not address the client's medical needs. Providing the client with tools to help her monitor diet (C) and identifying factors that enhance the problem (D) are outside the scope of practice of the therapist.

A 6-year-old is referred to therapy for treatment of separation anxiety. During the intake, the mother begins sobbing and shares that her husband recently left her for his secretary. The daughter appears very concerned as her mother talks. How would a psychodynamic therapist proceed? Response: A. Refocus the conversation on the needs of the 6-year-old. B. Provide psychoeducation to mother about the importance of her own mental health and how that can impact daughter. C. Engage in play therapy with the daughter during the session. D. Instruct the mother that she needs her own therapy.

B. Provide psychoeducation to mother about the importance of her own mental health and how that can impact daughter.

A 24-year-old young man walks into a therapist's office wearing a colorful headband, long purple scarf, and a fur coat. He states that he is happy with who he is, but he would like to better understand personal messages he receives from the newspaper and magazine articles. He adds that his strongly developed telepathic skills allow him to know what other people are thinking about him; thus he warns the therapist about the consequences of lying to him. What diagnosis should be considered initially? Response: A. Schizoid Personality Disorder B. Schizotypal Personality Disorder C. Delusional Disorder D. Histrionic Personality Disorder

B. Schizotypal Personality Disorder Rationale: This client is exhibiting odd, eccentric behavior, ideas of reference, and magical thinking. In addition, he is happy with who he is, which points to egosyntonic nature of the disorder. All of these symptoms are best accounted for by answer (B). Neither Schizoid PD, nor Histrionic PD have magical thinking or ideas of reference as their symptoms. Thus (A) and (D) are out. Since his behaviors are present across all aspects of his life and are egosyntonic, it cannot be Delusional Disorder (C).

A licensed therapist is leading a supervision group for interns at a community mental health clinic that serves a diverse population. The supervisor recognizes that the interns need support dealing with cultural awareness. In order to support the development of cultural competence, what step should the supervisor take first? Response: A. Provide information regarding clinical themes of different ethnic groups. B. Explore the interns' biases regarding different ethnic groups. C. Encourage interns to identify their own cultural heritage and world views. D. Teach interns techniques for exploring their clients' cultural influences.

C. Encourage interns to identify their own cultural heritage and world views.

A couple, Joe, age 39 and Marni, age 45, seeks therapy to determine whether they should stay together. They have been together for ten years but have never married. They report that they rarely argue but have a difficult time communicating. They deny any domestic violence or substance use problems. In formulating the initial treatment plan what should the therapist assess first? Response: A. How they handle conflict. B. Why they are seeking therapy. C. Their reasons for not getting married. D. Their views of potential separation.

A. How they handle conflict.

A 24-year-old client seeks treatment at a community mental health center. During the initial intake, the therapist notes that the client is dirty, confused, and appears to be responding to auditory hallucinations. How should the therapist proceed? Response: A. Obtain an informed consent and refer for a psychiatric evaluation. B. Complete a full mental status exam and initiate a 5150. C. Discuss the client's goals and psychiatric history. D. Contact the client's family members to complete the assessment.

B. Complete a full mental status exam and initiate a 5150.

A 83-year-old man has been in therapy for six months following the death of his wife. One session he informs the therapist that he recently changed his will, making the therapist the beneficiary of his estate. He states, "You have been so kind to me, you know me better than anyone." How should the therapist clinically manage the ethical issue presented in this vignette? Response: A. Inform the client that this is a boundary violation. B. Tell the client it is flattering but it is inappropriate due to the nature therapeutic relationship. C. Explain this would be a dual relationship. D. Assist him in finding a charity that could become the beneficiary.

B. Tell the client it is flattering but it is inappropriate due to the nature therapeutic relationship.

A 22-year-old client is referred by his lawyer after being accused of burglary. The lawyer claims that the client is depressed and wants him to be assessed and treated. When the therapist meets with the client, there are no symptoms of a mood disorder but there are signs of Antisocial Personality Disorder. How should the therapist proceed with treatment? Response: A. Inform the lawyer that he does not meet the criteria of depression. B. Refer the client to another therapist. C. Inform the client that he does not meet the criteria of depression. D. Inform the client that he may have Antisocial Personality Disorder.

C. Inform the client that he does not meet the criteria of depression.

A therapist is running a therapy group for adults with relationship issues. The group has been meeting weekly for two months. During a group therapy session, one of the members shares that he doesn't feel like he is getting anything from the group and is thinking of quitting. How should therapist respond? Response: A. Ask the client what the therapist could be doing differently. B. Identify ways the group has helped him. C. Invite the client to meet following group to discuss his concerns. D. Explore other group members' reactions.

D. Explore other group members' reactions.

A recently married couple, Marcus, an Asian American age 31, and John, a Caucasian age 28, are referred by Marcus's health insurance panel. John tearfully tells of Marcus's lack of affection over the past few months and adds that Marcus has been spending a lot of "extra time" at work with a new co-worker. Marcus shakes his head and says, "I'm tired of dealing with his paranoia. He's depressed! I am tired of him calling me at work and telling me he wants to die. I can't take it anymore." What ethical responsibilities does the therapist have based on the case provided in the vignette? Response: A. Manage the fee; Assess for John's suicide ideation; Review the limits of confidentiality. B. Manage confidentiality with insurance carrier; Inform couple that the therapist does not keep "secrets"; Identify expectations of treatment for both Marcus and John. C. Manage the fee; Assess for John's suicide ideation; Obtain consent to treat from the insurance provider. D. Inform the couple of the "no secrets" policy; Assess for John's suicide ideation; Identify expectations of treatment for both Marcus and John.

D. Inform the couple of the "no secrets" policy; Assess for John's suicide ideation; Identify expectations of treatment for both Marcus and John.

Brian, age 34, and Kim, age 30, are referred by Kim's co-worker for therapy. Kim shares that she recently caught Brian cheating on her and even though he says it's over, she doesn't know whether she trusts him. Brian says "I know I messed up. It won't happen again. I don't know why we have to keep dwelling on this." How should the therapist proceed with the initial assessment in this case? Response: A. Explore what conditions would allow Kim to trust Brian again. B. Invite the couple to discuss how they met and what keeps them together. C. Determine what prompted Brian to have an affair. D. Normalize Kim's feelings that she won't be able to trust Brian again. Score 0 of 1

B. Invite the couple to discuss how they met and what keeps them together.

A client who is going through a divorce and custody battle seeks Cognitive Behavioral therapy to help deal with the stress of his situation. He uses his insurance and the therapist bills for the sessions electronically. Three months into treatment, the client's records are subpoenaed and the client waives privilege. Because the therapist is HIPAA compliant, he must: Response: A. Obtain a signed waiver of privilege. B. Inform the client of any possible negative outcomes of releasing records. C. Release only the medical record. D. Release both the medical record and the psychotherapy notes.

C. Release only the medical record. Rationale: The correct answer is (C) as this relates specifically to being a HIPAA provider. Answer (A) must be done if you are HIPAA or not. (B) is not a condition of HIPAA and (D) is incorrect as you only turn over the medical record portion of the file.

A 26-year-old client who tested positive for HIV five months ago seeks therapy on the advice of his friend. During the fifth session the client shares that he's been "feeling less depressed and even went out to a bar and met new people for the first time in months." The client then admits his intention to not inform sexual partners of his diagnosis. How should the therapist clinically manage the ethical issues in this case? Response: A. Discuss AIDS prevention activities with the client. B. Encourage the client to inform the client's partners of the positive test. C. Inform the client that the therapist must inform the police because Tarasoff Duty to Warn has been triggered. D. Explore the client's resistance to sharing his diagnosis with his sexual partners.

D. Explore the client's resistance to sharing his diagnosis with his sexual partners

A 13-year-old boy is referred to therapy by his youth minister who shares that both of the boy's parents are "heavy drinkers". During the first session, the client shares that he is being harassed and pushed around at the school by a few of the older boys. He says he has told the teacher but that only made it worse. How should the therapist proceed clinically in this case? Response: A. Contact his parents and inform them of the bullying. B. Assess for child abuse. C. Refer the client to a social skills group. D. Work with client to identify safety strategies.

D. Work with client to identify safety strategies.

Tamara, a 38-year-old Baptist minister, and her 28-year-old husband, Michael, who are both African American, self-refer for couples therapy. Tamara recently gave birth to a son with Downs Syndrome. Tamara states, "Michael doesn't understand my stress. I have a congregation that relies on me, but I am so worried about everything that I can't sleep and can't think clearly." She complains that Michael drinks nightly and doesn't get out of bed all day. "I can't trust him with our baby." Michael responds, "I begged you not to go through with the pregnancy when we found out the baby was defective. Don't blame me for your child care problems while you keep working 16 hours every day." Throughout the initial appointment, Tamara compulsively checks her cell phone for messages, while Michael sits rigidly with his arms crossed and stares at the floor. How would a Cognitive-Behavioral therapist incorporate the couple's religious beliefs into the treatment plan based on the case presented in the vignette? Response: A. Enable the couple to identify the positive value of the marital dyad as embraced by their religious beliefs; encourage Tamara and Michael to explore alternative spiritual interpretations for their marital discord. B. Assist the couple in using their religious beliefs to reframe the perception of the child as "defective"; facilitate dialogue between Tamara and Michael about the impact of Tamara's ministry on their marital discord. C. Explore the thoughts that trigger Michael's desire to drink; promote dialogue on how potential divorce would be received within the religious community. D. Recommend that Tamara and Michael explore alternative spiritual interpretations for their marital discord; challenge the couple's insight into their spiritual differences regarding their baby.

B. Assist the couple in using their religious beliefs to reframe the perception of the child as "defective"; facilitate dialogue between Tamara and Michael about the impact of Tamara's ministry on their marital discord. Rationale: Answer (A) is wrong since it doesn't specifically include CBT interventions and neither part of the answer is indicated in the stem. The value of the marital dyad is not being questioned in the stem, and there is no need to explore alternative interpretations of the marital dyad, as much as there is a need to determine the couple's understanding of the marital discord. Answer (B) has all the necessary parts to it. It describes CBT interventions that are relevant to the case, and religious beliefs are incorporated. Answer (C) is out since the first part only deals with Michael and ignores the couple, and the second part brings up a divorce, which was not mentioned in the stem. The first part of answer (D) has already been discussed, and the second part is not appropriate since a CBT therapist is not going to challenge the couple's insight but will teach the couple how to challenge their own thoughts.

A client with a history of anger issues and aggressive behavior has been seeing a therapist for therapy for two months. During a heated session, the client storms out. The following day he delivers a letter demanding his records or he will take legal action. The therapist believes that releasing his records will cause him serious harm. What action should the therapist take FIRST in this case? Response: A. Seek legal consultation regarding the threat to sue. B. Document in the record both the request and the rationale for non-release. C. Develop a safety plan knowing that the client will not be happy with the outcome. D. Schedule a session to discuss the client's desire to obtain records.

B. Document in the record both the request and the rationale for non-release.

A 58-year-old woman is referred to therapy by her oncologist. The client was recently diagnosed with breast cancer. While her prognosis is good, the client has become very depressed and withdrawn. She shares with the therapist that the cancer has forced her to look at her life and she doesn't like what she sees. She admits that she was a workaholic when her two children were young and now has little contact with them. She also shares that she feels like her husband is a stranger. What intervention would an Existential therapist employ during the middle phase of treatment? Response: A. Explore the client's understanding of her early relationships and how they impact her current situation. B. Encourage the client to "play out" scenarios of choices she faces and acknowledge the accompanying fears and anxieties. C. Teach the client communication skills to better engage with her family members. D. Identify the underlying thoughts and beliefs that lead to her troubling feelings.

B. Encourage the client to "play out" scenarios of choices she faces and acknowledge the accompanying fears and anxieties.

A teacher consults with a school-based therapist regarding a six-year-old student who has repeatedly come to school with the same injuries. The student tells the teacher the injuries are a result of playing with the family's dog and participating in outdoor sports. The teacher reports the student is increasingly distracted during class, spending more time alone, being aggressive with classmates, and is reluctant to leave the classroom at the end of the day. What should the therapist do next? Response: A. Discuss the client's injuries with the school nurse. B. Explain to the teacher the need to report abuse. C. Contact the parents to inquire about the child's unusual behaviors. D. Meet with the student to complete an assessment.

B. Explain to the teacher the need to report abuse. Rationale: The information provided in the stem is enough to reasonably suspect child abuse. It is the responsibility of a mandated reporter to notify the appropriate authorities when abuse is suspected (B). Both the teacher and the therapist would not need to make a report. In this case, the teacher would make a report, since he/she has witnessed the child's behavior first hand and the therapist would document that such report was made. It is the responsibility of the appropriate authorities to investigate whether abuse exists. So (A), (C), and (D) are out.

A 32-year-old client seeks therapy to address relationship issues. She has two young children and recently left an abusive relationship. She was married for 7 years and states that the last 5 years included verbal and emotional abuse. She reports depressive symptoms, trouble in dealing with separation, low self-esteem and difficulty parenting her children. What interventions would a Structural Family therapist utilize in the middle phase of treatment to strengthen the parental hierarchy between the client and her children? Response: A. Initiate enactments to help client identify specific ways to enact clear boundaries. B. Have client arrange chairs to represent current hierarchy and rearrange chairs to represent her status as a parent. C. Track and match client's interactional style. D. Identify the influence and control her spouse has had on her in the past.

B. Have client arrange chairs to represent current hierarchy and rearrange chairs to represent her status as a parent.

Joan is a 35-year-old single woman who is referred by her EAP. She states she was recently transferred to a new department and was given a "write-up." She complains, "My old supervisor was threatened by me. This always happens—I work hard and then get penalized for it! People in charge always think they know better." Joan shakes her head and goes on, "You really can't trust anyone these days." What factors in Joan's personal history need to be considered to establish a therapeutic alliance in this case? Response: A. Joan's relationship difficulties may diminish her capacity for trust. B. Joan's history of difficulties with authority may cause her to reject help. C. Joan's anxiety may block her communicating in an unguarded manner. D. Joan's history of misinterpreting the motives of others may contaminate the therapy.

B. Joan's history of difficulties with authority may cause her to reject help.

Kevin, a 48-year-old male, is in treatment for issues of depression. At the end of the sixth session, Kevin angrily states that his 18-year-old stepson is getting on his nerves and it is time Kevin showed him "who is the man in the house." Two days after the session, the therapist receives a phone call from Kevin's wife. She is concerned because Kevin left to go meet her son at the cabin in the woods for a "stern talking to." His wife states that Kevin was very irritable when he left the house. The therapist's responsibility is to: Response: A. Encourage Kevin's wife to call 9-1-1. B. Maintain confidentiality. C. Call the police and warn Kevin's step-son. D. Contact Kevin and find out whether he has any means of hurting his step-son.

B. Maintain confidentiality. Rationale: The therapist's responsibility in this case is to maintain confidentiality of the client. Kevin's wife is not giving us enough information to substantiate Tarasoff. Therefore, on the phone with her, we are going to maintain confidentiality and not even acknowledge to her that Kevin is our client (B). Based on the information provided in the stem, the threat of harm is unclear. Encouraging the wife to call 911 (A), calling the police and warning Kevin's step son (C), or contacting Kevin to find out whether he has means of hurting step son (D) are overreactions based on the information provided.

Maricel, a 37-year-old single woman, is referred by her mother's doctor. Maricel is a high school math teacher. She was born in the Philippines but moved to the United States when she was two. Recently, her elderly mother suffered a stroke and is currently in a coma on life support. Maricel has been the primary caregiver for her mother since her father died two years ago. Maricel has decided that she wants to terminate life support. However, her uncle, who is her mother's brother, is against it. Tearfully, Maricel shares, "In front of the doctor, he says that I have the final say, but in private he is against me doing it. I'm sure that all his siblings are going to take his side in this matter. He doesn't care that this is what my mom wanted." She adds, "I just can't concentrate at work; I'm snapping at my co-workers and friends and then I feel awful about it. I'm crying all the time, not sleeping, and I'm tense. I just don't know what to do. I don't want to get out of bed in the morning." Which of the following human diversity issues should the therapist consider when developing the comprehensive treatment plan in the case presented in the vignette? Response: A. Maricel's cultural and religious views of death and dying and the impact of being a single woman and sole caregiver of mother. B. Maricel's spiritual beliefs regarding end-of-life issues and the influence her Filipino culture plays in this situation. C. Maricel's departure from traditional Filipino roles and the impact of being a single woman and sole caregiver of mother. D. Filipino attitudes towards Western medicine and the role of religion and spirituality in Maricel's family.

B. Maricel's spiritual beliefs regarding end-of-life issues and the influence her Filipino culture plays in this situation. Rationale: When developing a treatment plan, it is important to take into consideration matters that are both relevant to the presenting issues and that address diversity issues, making (B) the best answer. Maricel's spiritual beliefs regarding end-of-life issues are important to take into consideration, since the presenting complaint is her mom's dying wishes. In addition, what sets this case apart from other similar cases is the fact that Maricel is heavily influenced by her culture and her Filipino family in making this decision. Answer (A) is not the best answer because we do not see evidence of Maricel's religious views in the stem or that her being a single woman is somehow impacting her decision making process in this case. Answer (C) is out since there is no evidence in the stem that points to the fact that Maricel has departed from traditional Filipino roles. There are no indicators of answer (D) in the stem either, as there is no mention of Filipino attitudes towards Western medicine or the role of religion in Maricel's caregiving.

57-year-old Nolan and 54-year-old Luisa seek treatment due to marital discord. Luisa complains "Nolan is always quiet when he gets home from work. It's as if we have nothing to talk about, nothing in common." Nolan utters "Ever since the kids went off to college, all Luisa does is rearrange their rooms and tries to rent them out on Airbnb. She is bored and disinterested with me." How should the therapist best proceed during the first session? Response: A. Conduct an inventory of the couple's attachment style. B.Provide psychoeducation to the couple about the life stages. C. Ask Nolan and Luisa to put themselves into the shoes of their partner and describe their experience. D. Schedule individual sessions with each member of the couple.

B. Provide psychoeducation to the couple about the life stages. Rationale: The first thing for the therapist to do when the couple presents with a life stage issue is to provide psychoeducation and normalize their feelings (B). All other interventions might come into play once the source of conflict is understood and proper assessment is completed.

A therapist in private practice has been treating a 54-year-old male client for four months. Over the past two months, the client regularly arrives late to appointments and has also missed several of them. Although the cancellation policy was clearly explained at the onset of therapy, the client continuously fails to call in advance to cancel. When confronted, the client minimizes the problem and accuses the therapist of having unreasonable expectations. How should the therapist initially respond to the client? Response: A. Remind client of his cancellation policies, have him sign an attendance contract and require he pay for missed appointments. B. Confront the client about his perception of the problem and discuss how to move forward. C. Reframe client's behavior as having difficulty with therapeutic work and collaborate with client to reformulate goals. D. Review cancellation policy and discuss possible termination with client.

B. Confront the client about his perception of the problem and discuss how to move forward.

A therapist meets with a young woman who is in her first year of college. She expresses feelings of depression and loneliness. She had a close group of friends in high school, played sports, and received straight As. She has met several people but has not developed many good friendships and her grades are low. She says, "If things don't get better, I'm not sure what I'll do." What should the therapist do first? Response: A. Help client prioritize her goals and identify obstacles. B. Rule out thoughts of self-injury. C. Refer her to a psychiatrist for a medication evaluation. D. Explore client's healthy coping skills.

B. Rule out thoughts of self-injury. Rationale: This question asks what the therapist should do first. The therapist's priority in this situation is to monitor and manage the client's safety by first asking the client if she has any thoughts of hurting herself (B). The therapist may choose to use the interventions identified in answers (A), (C), and (D) but only after assessing safety.

A long-term client being treated for major depression shares that she is pregnant and that she intends to stop taking her medication to avoid the possible negative effects to the fetus. The therapist is concerned because the client became severely depressed the last time she went off her medications. The therapist should: Response: A. Encourage client to consider staying on medications. B. Contact the psychiatrist and share concerns. C. Inform the client that the possible side effects outweigh the risk of going into a depression. D. Manage countertransference by consulting with a colleague.

B. Contact the psychiatrist and share concerns. Rationale: The therapist should consult with the psychiatrist to voice concerns and coordinate care, so B is the best answer. This could lead to answer A. Answer C is outside of the scope of practice and should come from her MD. This isn't an issue of countertransference as it is a valid clinical concern so D is out.

Dan, age 38, and his wife Mary, age 35, are referred by Dan's oncologist. Dan shares that he was recently diagnosed with a brain tumor that is inoperable and the doctors say he has three months to live. They share that they haven't told their two teenage daughters and are seeking therapy to help them all cope. Mary begins to cry and says, "I know my older daughter is going to be devastated. You can help her right?" What legal obligations does the therapist have initially in the case provided in the vignette? Response: A. Explore their expectations for treatment; Determine whether Dan has an Advanced Directive; Work within scope of practice. B. Obtain informed consent for treatment; Maintain separate files if family members are seen individually; Obtain a signed release before consulting with Dan's doctor. C. Encourage the family to create an advanced directive for Dan; Work within scope of practice; Obtain a release before consulting with Dan's doctor. D. Obtain informed consent for treatment; Obtain a release before consulting with Mary's doctor; Set fee before onset of therapy.

B. Obtain informed consent for treatment; Maintain separate files if family members are seen individually; Obtain a signed release before consulting with Dan's doctor.

A 52-year-old woman bring her 86-year-old mother to therapy. The woman reports that her mother has seemed depressed since she relocated from another state and moved into an assisted living facility nearby. The mother confirms that she's been feeling really down since the move but denies any suicidal ideation. What approach should the therapist consider in this case? Response: A. Psychodynamic therapy to understand her relationship patterns B. Reminiscence therapy to review her life C. Cognitive therapy to confront her negative thought patterns D. System therapy to improve her communication with her family

B. Reminiscence therapy to review her life

A therapist in private practice is on three different insurance panels. The therapist receives notification that one of the insurance companies is conducting an audit and they request six clients' records. What should the therapist do? Response: A. Determine the reason for the audit. B. Make sure each file has an authorization and submit the records. C. Request written authorization for each file and inform clients about information released. D. Provide copies of the records.

B. Make sure each file has an authorization and submit the records. Rationale: The best answer is B. Audits are treated as part of the federal HIPAA exception permitting access to health records for treatment, payment, and health care operations (TPO) activities without an authorization from the client. If you are on an insurance panel, you have agreed to allow your records to be audited. The clients, when they agree to utilize their insurance and sign HIPAA forms, also agree in advance to release records for insurance audits. However, the therapist should verify the authorization is signed and in record before releasing the file.

A recent Chinese immigrant is seen in a hospital emergency room after several panic attacks. He is referred to a therapist but declines mental health treatment. All of the following are likely explanations for this refusal except: Response: A. High incidence of anxiety disorder within the Chinese culture. B. Possible language barriers. C. Alternative treatments available in the culture. D. Cultural stigmas associated with treatment.

A. High incidence of anxiety disorder within the Chinese culture. Rationale: The correct answer is (A) since there is no evidence to support the fact that anxiety is highly prevalent in Chinese culture. Even if it is, it would not provide an adequate explanation as to why the client is refusing treatment, versus options (B), (C) and (D), which are all possible explanations for such refusal.

A family is referred for therapy after the adoption of their 8-year-old nephew. The adopted child's mother was incarcerated for selling drugs and the father is deceased. The parents report that that child hoards food, is frequently aggressive with their other children ages, 9 and 6, and tends to reject their efforts to include or console him. What long-term goal should the therapist consider with this family? Response: A. Improve adopted son's self-confidence and impulse control. B. Establish and maintain healthy family connections. C. Resolve feelings of loss and abandonment. D. Identify family reactions regarding recent adoption.

B. Establish and maintain healthy family connections. Rationale: The question wants the "long-term goal" for the family so B is the best answer that addresses the whole unit and would be what the therapy should work toward. A and C are short term goals that could be addressed initially. D is an initial intervention that could be used.

A therapist at a community mental health center has been working with an El Salvadorian family to address issues with their teenage children. At the beginning of the final session, the mother gives the therapist a gift card and expresses her appreciation to the therapist for continued support of the family. How should the therapist address the gift with the client? Response: A. Acknowledge the expressed gratitude and inform the client that you cannot accept gift cards. B. Determine the value of the gift prior to accepting it. C. Consult with a colleague who is knowledgeable about El Salvadorian culture. D. Explain to the family that it is your agency's policy not to accept gifts from clients.

A. Acknowledge the expressed gratitude and inform the client that you cannot accept gift cards. Rationale: Gift cards are equivalent to cash and that would be an inappropriate gift to accept, so the best answer is A. You don't need to find out the value or consult with a colleague.

During an initial session, a mother tells a therapist that her three year-old son is stubborn, self-centered, doesn't listen, and refuses to help her until she starts doing something, and then he wants to help. He tantrums when she tries to show him how to do things properly and insists on doing it his way. The therapist's best initial response would be to: Response: A. Acknowledge/explore mothers feelings around parenting struggles and educate the mother about child's development. B. Schedule a biopsychosocial evaluation of the child and refer mom to a parenting group for parents of strong-willed children. C. Review possible behavior modification techniques and examine the mother's expectations of parenting. D. Provide play therapy for the child and and joint play therapy for mother and child.

A. Acknowledge/explore mothers feelings around parenting struggles and educate the mother about child's development. Rationale: The therapist's best response would be to acknowledge and explore the mother's feelings and provide the mother with education about normal development (A) since the symptoms described appear to be signs of normal child development. If, after receiving information regarding normal development, the mother continues to express concern or identifies behavior outside the norm, it could be appropriate for the therapist to schedule an evaluation (B), review behavioral techniques (C), or provide play therapy (D).

A 38-year-old medical doctor seeks treatment for depression. The client states that her whole life she had worked to become a doctor and now that she was finally practicing independently she was miserable. She shares that she recently broke up with her boyfriend and says she's afraid she'll never meet someone who can measure up to her expectations. In the initial phase of treatment, what interventions would an Adlerian therapist utilize compared to a Cognitive Behavioral therapist? Response: A. An Adlerian therapist would explore the client's style of life and early recollections; a Cognitive Behavioral therapist would help the client identify her problematic beliefs. B. An Adlerian therapist would explore what kind of life the client would like to have; a Cognitive Behavioral therapist provide psychoeducation on CBT. C. An Adlerian therapist would explore the client's level of social involvement; a Cognitive Behavioral therapist would help the client identify her problematic belief. D. An Adlerian therapist would complete a family constellation; a Cognitive Behavioral therapist explore the origins of the client's problematic beliefs.

A. An Adlerian therapist would explore the client's style of life and early recollections; a Cognitive Behavioral therapist would help the client identify her problematic beliefs. Rationale: Both parts of answer (A) correlate with the theories in question and both are initial stage interventions that are relevant to this case. The first half of answer (B) is descriptive of a Solution Focused theory, so (B) is out. Answer (C) is wrong since exploring the client's level of social involvement is going to be part of exploring client's style of life, and exploring the client's early recollection is crucial for Adlerian therapy, so the first part of answer (A) is more comprehensive. The second part of answer (D) rules it out, since CBT therapists do not explore the origins of problematic beliefs.

39-year-old Emilio and 36-year-old Helena are referred by their middle child's teacher. They have 2 sons, ages 11 and 7, and a 9-year-old daughter, Raquel. Both parents disclose being very busy and stressed at work, which means that neither of them is able to be home much. They are both concerned about their daughter Raquel, who appears more and more withdrawn and shut down lately. "I can't get her to do anything", Helena complains. "Plus she used to love going over to her aunt's and uncle's place, but, ever since a couple of months ago, she refuses to set foot in that house. I wonder if it has anything to do with that incident?" How would a therapist assess and clinically manage potential child abuse in this case? Response: A. Ask Helena what she means by "that incident"; inquire about any other recent changes in Raquel's behavior; process the parents' reactions to a possible child abuse report. B. Review parenting skills modeled by the couple's own parents; file a child abuse report based on the information provided; process the parents' reactions to filing a child abuse report. C. Evaluate current child care arrangements; obtain a release to speak with Raquel's teacher; file a child abuse report based on Helena's concerns. D. Obtain a release to speak with Raquel's teacher; ask Helena what she means by "that incident"; explain the therapist's obligation to report child neglect.

A. Ask Helena what she means by "that incident"; inquire about any other recent changes in Raquel's behavior; process the parents' reactions to a possible child abuse report.

William, age 59, brings his daughter Terrie, age 15, for therapy. William says, "Terrie has been cutting for a year, cries in her bedroom, and worries about everything." William's wife, Florence, age 58, is a Scientologist and against psychotherapy. She's been treating Terrie with homeopathic supplements and exercise. Terrie shares that she thinks about death and tried to hang herself a few years ago. She shows you the scars and fresh cuts on her right forearm. Terrie says, "I show my parents these and they do nothing about it, even when the cuts are deep and bleeding really badly." Terrie begins to cry and says to her dad angrily, "I don't want to be homeschooled! You guys shouldn't have adopted us. All you care about is your business." William shakes his head and says, "I'm too old to deal with this. How much is this all going to cost?" How would a Systems therapist incorporate human diversity into the treatment plan based on the case presented in the vignette? Response: A. Ask William and Terrie to tell you how Florence's religious beliefs impact their family; explore how being older parents impacts the different family members; explore their experience of being an adoptive family and how it affects their understanding of the situation B. Remove Terrie's position of the identified patient by confronting Florence's religious beliefs; evaluate the validity of Terrie's beliefs that her parents do not care about her; explore their experience of being an adoptive family and how it affects their understanding of the situation C. Educate them on how being an adoptive family may affect their understanding of the situation; Explore Florence's religious beliefs and how they affect Terrie's treatment; discuss how the other family members are impacted by Terrie's problems D. Explore how being older parents affects William's view of the family's issues; ask William and Terrie to tell you how Florence's religious beliefs impact Terrie's illness; determine whether homeschooling is the best option for Terrie

A. Ask William and Terrie to tell you how Florence's religious beliefs impact their family; explore how being older parents impacts the different family members; explore their experience of being an adoptive family and how it affects their understanding of the situation Rationale: The right answer for this question needs to respond to the issues presented, be consistent with the concepts of System's theory, and include issues of human diversity that are playing out in this case and are unique to this family. Answer (A) includes all of these components. The parts of the answer are geared towards all family members and include words like "explore" and "ask", which is consistent with System's theory. Florence's religious beliefs are certainly influential in this case as they are rather specific. Being older parents as well as an adoptive family has its own set of influences that not all families are dealing with. Answer (B) is wrong since a System's therapist is not going to confront Florence's religious beliefs. Doing so will not help to remove Terri's position of an IP. The second part of answer (B) only addresses Terry and a System's therapist would address the dynamic of the whole family. Answer (C) is not the best answer, since it has the therapist educating the family on the impact of being an adoptive family, as opposed to exploring their view on how it is impacting the situation. The family is going to know more about their experience than the therapist would. Answer (D) is not fitting since the first half deals only with William and leaves the rest of the family out. The second half of answer (D) targets Terrie as an IP, something a Systems therapist would avoid doing. In addition, the therapist is not going to determine whether homeschooling is the best option; that would be imposing the therapist's views onto the family.

A lesbian couple seeks pre-marital counseling. The women, one a 36-year old African American high school teacher and the other a 34-year-old Caucasian graphic design artist, have been dating for three years and living together for the past year. When the therapist inquires about their reasons for seeking therapy, the teacher shares in a frustrated tone that their only issue is that the other woman smokes marijuana daily and refuses to quit despite promising to do so. In response to this, the other woman groans and says, "I don't understand why this is such an issue for you." What actions should the therapist take first? Response: A. Ask the woman who uses marijuana to tell the therapist about the role marijuana plays in her life. B. Explore with the woman who uses marijuana when the problem with marijuana started and her views on how it has impacted their relationship. C. Reflect back to the teacher that marijuana seems to be causing them a lot of problems. D. Determine whether the teacher has ever used marijuana and how her use or non-use impacts their situation.

A. Ask the woman who uses marijuana to tell the therapist about the role marijuana plays in her life. Rationale: The best answer is (A). Answers (B) and (C) both focus on the marijuana use as a "problem" instead of exploring it from a neutral stance. One client does not see it as a problem, so using that wording can be harmful to the therapeutic relationship. This would make the therapist seem to be siding with the teacher. Answer (D) could be done at some point but doesn't address the main presenting problem. Answer (A) is a technique from motivational interviewing that explores the role the substance plays in the person's life instead of pushing them to see it as a problem.

A young boy is brought in by his parents to therapy. The 6-year-old boy has had several surgeries to correct a birth defect. The parents report the child is traumatized by his hospital experiences and expresses fear of going to the hospital or the doctor. As a psychodynamic therapist working with this child, what would a middle phase treatment goal be: Response: A. Child can recall traumatic event without becoming overwhelmed with negative emotions. B. Child is able to go to the doctor with the support of the therapist and parents. C. Child is able to display a full range of emotions without experiencing a loss of control. D. Child is able to go to the doctors office without exhibiting any resistance.

A. Child can recall traumatic event without becoming overwhelmed with negative emotions. Rationale: (A) is the correct answer—that as the therapist works with the client there is a reduction in affect when recalling the traumatic event. (B) is out because typically the therapist would not accompany the child to the doctor. (C) is a goal for the end of treatment and this answer does not address the hospital/doctor issue that the child is working on. (D) is not a realistic goal, since it might be healthy for the child to exhibit some resistance in these circumstances.

A 28-year-old veteran of the Iraqi war comes to therapy for PTSD symptoms. She reports having flashbacks and nightmares not only from being in direct combat zones but also from being sexually harassed by the men in her platoon over the two years she was deployed. She is currently dropping out of community college because of her symptoms. After 3 months of therapy she is finding relaxation techniques helpful when she feels triggered. How should the therapist proceed with treatment in this case? Response: A. Continue therapy; Participate in Prolonged Exposure Therapy to reduce fear and avoidance associated with trauma; Teach mindfulness meditation. B. Refer to an EMDR therapist; Re-apply to College program; Refer to group for sexually assaulted women. C. Teach thought stopping techniques; Share trauma stories in detail; Encourage her to press charges against the men who harassed her. D. Continue therapy; Mobilize the client's support system; Refer to a veteran's support group.

A. Continue therapy; Participate in Prolonged Exposure Therapy to reduce fear and avoidance associated with trauma; Teach mindfulness meditation.

A therapist in private practice meets with a new client who shares that she is also a therapist. The client discloses that she has been practicing for over twenty years and is considering closing her practice. She shares that over the past year she developed feelings for a client and had a brief affair, but ended it because she knew it was wrong. She indicates feelings of guilt and remorse and feels she should no longer practice because of her actions. How should the therapist manage the ethical responsibilities presented in this case? Response: A. Explore the client's feelings of guilt and remorse, as well as her thoughts about ending her practice. B. Validate her feelings of guilt and remorse and provide her with the brochure "Professional Therapy Never Includes Sex." C. Encourage the client to obtain legal counsel. D. Encourage the client to address the ethical violation directly with her client and provide him with the brochure, "Professional Therapy Never Includes Sex."

A. Explore the client's feelings of guilt and remorse, as well as her thoughts about ending her practice.

A therapist sees a college-aged client who was raised in a family with an alcoholic mother who was abusive toward the client and her siblings. The client tells the therapist that she and her siblings regularly avoid getting angry or upset around her mother, because they are afraid of how she will react. The client also says she tries to keep her mother happy and that she feels this is the only way for her to be happy. The therapist's short-term goal would be to: Response: A. Help the client understand characteristics of codependency. B. Assist client in expressing her feelings to her mother. C. Help client access and express her anger. D. Prepare the client for family therapy.

A. Help the client understand characteristics of codependency. Rationale: The stem describes a codependent situation, so the therapist needs to first help the client understand this behavior (A). Assisting the client in expressing her feelings (B) or helping the client access and express her anger (C) address only one element of the broader issue. Family therapy (D) is not a realistic option at this point.

Sylvia, a 68-year-old woman, is referred by her priest from her church after a below-the-knee amputation due to complications from diabetes. Sylvia begins to cry and shares that "life is too hard without her leg." She lives alone and her daughter visits her once a week but she lives an hour away. She tells the therapist that her doctor wants her to take medicine for depression but she is scared. She adds, "I don't think this talking stuff is going to help me." What factors require primary consideration by the therapist in formulating the initial treatment plan? Response: A. Premorbid coping skills; Sylvia's motivation for treatment; Sylvia's religious community B. Sylvia's feelings of shame; Sylvia's motivation for treatment; Sylvia's lack social support system C. Premorbid coping skills; Sylvia's psychiatric history; Sylvia's motivation for treatment D. Sylvia's substance use; Sylvia's adaptation with activities of daily living; Sylvia's lack of family support

A. Premorbid coping skills; Sylvia's motivation for treatment; Sylvia's religious community

A therapist working in a mental health hospital sees a 67-year-old involuntary client who is being held due to the risk of self-harm. The client has heart disease and a history of cocaine abuse. The client does not feel she needs to be in the hospital and requests the therapist transfer her to an outpatient program. How should the therapist initially respond? Response: A. Remind the client that hospitalization is not voluntary. B. Assist the client in efforts to be discharged and locate outpatient program. C. Explain that the client will be released after 72 hours. D. Refer the client for an outpatient evaluation.

A. Remind the client that hospitalization is not voluntary. Rationale: Clients who are involuntary hospitalized are not able to determine when they are to be released and should be reminded of this if attempting to receive alternative treatment (A). The therapist does not have the authority to discharge the client (B) and (D) nor determine when the client will be released (C).

A 17-year-old is referred by her doctor to a mental health agency for counseling. The client learned several weeks ago that she's pregnant. She reports she is in a relationship with the person who got her pregnant, but they are not getting along, she does not want him to know, and does not think she wants to have the baby. She is conflicted about what to do and tells the therapist she would like support as she decides what to do. The therapist has personal beliefs against abortion and feels strongly the client should not have an abortion. How should the therapist address the conflict presented in this case? Response: A. Seek immediate consultation to discuss the matter and determine if a referral would be appropriate. B. Seek consultation and refer the client to prenatal services. C. Encourage the client to include her boyfriend in making a decision. D. Share the therapist's personal beliefs and process how it effects the client and therapist's relationship.

A. Seek immediate consultation to discuss the matter and determine if a referral would be appropriate.

A therapist working in the psychiatric hospital is asked to complete a mental status exam of a newly admitted patient. The patient is confused and unable to provide intake information during the hospital admission. What should the therapist do next? Response: A. Seek permission from the patient to contact relatives. B. Recommend psychological testing for the client. C. Discuss the case with the therapist's supervisor. D. Wait until the client is stable before completing the report.

A. Seek permission from the patient to contact relatives. Rationale: People are admitted to the hospital for serious reasons: they might have tried to attempt suicide by taking some substance or might be a danger to self or other in some other way. If the client is unable to provide intake information, the next best source of information could be family or significant friends (A). Seeking permission to make these contacts would be an appropriate and ethical thing to do. The client might not be in the clear enough state of mind in order to give us a release to talk to the relatives, but that does not mean that we wouldn't try.

Dan, age 46, his girlfriend Nicole, age 34, and her daughter Allison, age 16, are referred to therapy by Allison's school. She has been isolating herself and crying a lot since a cross-country move last month. Dan angrily says, "I'm out of a job and we might lose our house. Allison needs to toughen up and stop complaining about changing schools and missing her friends." Nicole, who has a split lip, timidly explains that she thinks Allison is upset because she and Dan aren't getting along. Allison does not make eye contact and begins to cry softly and says, "Dan drinks too much. I am tired of hearing them fighting. I can't take it anymore." What interventions would a Solution Focused therapist use to treat Allison in the middle phase of treatment? Response: A. Validate what Allison is doing well to cope with her problems and identify on a scale of 1-10 how Allison feels she has been coping with her situation. B. Encourage Allison to do more of what has been working for her and teach communication skills so she can make new friends at school. C. Find out what would be different if things in Allison's life were better and encourage Allison to do more of what has been working for her. D. Identify on a scale of 1-10 how bad Allison feels her problems are and validate what Allison is doing well to cope with her problems.

A. Validate what Allison is doing well to cope with her problems and identify on a scale of 1-10 how Allison feels she has been coping with her situation.

The parents of a 15-year-old boy consult with an employee assistance program therapist regarding their child's shift in behavior since entering high school. During the past year his grades have dropped dramatically, and he was recently placed on juvenile probation after being caught attempting to steal alcohol from a grocery store. He was adopted when he was two years old after he had been in several foster homes. Before this last year his behavior and development had progressed normally. The therapist should: Response: A. refer the entire family for assessment and treatment. B. see the adolescent and his family for brief therapy. C. educate parents regarding normal adolescent behavior. D. assess for physical abuse or neglect due to marked change in behavior.

A. refer the entire family for assessment and treatment. Rationale: Some of the behavior described in the stem are extreme and do not reflect normal adolescent development (C). The appropriateness of brief therapy (B) could only be considered after an assessment is conducted. There is not enough information in the stem to suspect abuse or neglect (D), since sudden shift in behavior can be accounted for by substance use or a medical issue. Thus, due to severity of the boy's behaviors, further assessment and appropriate treatment are indicated (A).

Ana, a Mexican-American 39-year-old single mother of two young children, is referred by her minister. Ana shares that she has been very overwhelmed lately. Her parents were visiting for three months but recently went back to Mexico. She says, "They really helped me with the kids. Childcare is so expensive and my kids are in different schools now. My boss at work is getting irritated that I have been late a lot. I am worried I am going to lose my job." She adds that she's been having difficulty sleeping and bad headaches. What human diversity issues should the therapist consider when developing an initial treatment plan in this case? Response: A. Ana's health concerns; Her low socioecomic status; Being a single mother. B. Ana's spiritual beliefs; Her experience as a Mexican-American; Being a single mother. C. Ana's spiritual beliefs; Her low socioeconomic status; Being a single mother. D. Ana's sexual orientation; Being a single mother; Her experience as a Mexican-American.

B. Ana's spiritual beliefs; Her experience as a Mexican-American; Being a single mother. Rationale: In answering this question we need to think about this client's unique life circumstances that are shaping her presenting issues. In the stem we see that she is a Mexican-American, is a single mother, and she was referred by her minister. All three of these things set her aside from other people with similar presenting issues, thus (B) is the best answer. Answer (A) is out since lots of people share similar health concerns and that is not a human diversity issue. Plus, we would be making an assumption if we were to state that her low socioeconomic status is at play, since the stem doesn't explicitly state that she has a low socioeconomic status. Answer (C) is also out due to the assumptions regarding her low socioeconomic status. And (D) is wrong because we do not know her sexual orientation.

A therapist meets with a client who describes feelings of depression that began soon after her partner moved into the home. The client replies with a flat "fine" when asked about the new arrangement, but also mentions offhandedly that she has to go straight home after the session because the partner monitors the car mileage and time spent away from home. The client seems tense. The therapist should? Response: A. Suggest the benefits of a joint session. B. Explore how the couple deals with conflict. C. Recommend that she has a frank discussion with her partner. D. Provide information about domestic violence.

B. Explore how the couple deals with conflict. Rationale: The therapist's best course of action would be to explore how the couple deals with conflict (B). This would allow the therapist to gather additional information to determine if domestic violence is present in the relationship. The therapist should not suggest a joint session (A) or recommend the client have a frank discussion with her partner (C) if there is a likelihood of domestic violence. The therapist could provide information about domestic violence (D), but only after determining it is appropriate for the client.

A therapist in private practice is conducting a weekly support group for postpartum mothers. During a group discussion about the challenges new mothers confront, one of the mothers begins to sob and says she is struggling to get through each day. She notes that she has lost interest in daily activities, spends most of the day in bed and feels disconnected from her husband and baby. How should the therapist clinically manage the client's disclosure? Response: A. Provide psychoeducation to group about postpartum blues; explore if other group members feel the same; refer to individual therapy. B. Meet with client after group: assess for suicide; refer to individual therapy. C. Normalize the client's experience; explore if other group members feel the same; refer to individual therapy. D. Provide psychoeducation to group about postpartum blues; help client identify supports; explore ways to improve connection to family.

B. Meet with client after group: assess for suicide; refer to individual therapy. Rationale: What can we eliminate? (A) and (D) are out because this is more than "blues." That leaves (B) and (C). (B) is the best because it meets with the client privately, assesses for risk, and refers her to individual therapy which is indicated. (C) is problematic because it normalizes something that, while common, is not "normal." It also invites others to say if they have the same thing and there is a good chance that no one else is feeling as bad as this; so it could increase the client's shame. (B) is the better option as it responds to the severity of the woman's complaint.

A 6-year-old, Nate, is referred to therapy by his teacher because he is very disruptive in class, uses obscene language, fights with other students and consistently speaks out of turn. The parents, Steve and Nancy, corroborate that he shows similar behavior at home and does not respond to punishment. The parents add that their son seems unresolved in his grief over his baby sister's recent death. In the initial phase of treatment, how would a Narrative therapist intervene compared to a Solution Focused therapist? Response: A. Narrative would identify times when Nate's behavior wasn't a problem at school and/or at home and provide psychoeducation on grief in children; Solution Focused would normalize Nate's behavior and identify family strengths. B. Narrative would separate Nate from his bad behavior and explore the effects of the bad behavior at school and home; Solution Focused would ask about times when Nate had behaved well and how the family would know the problem was gone. C. Narrative would provide psychoeducation on grief and explore how the family has attempted to punish Nate in the past; Solution Focused would identify family strengths and unique outcomes when the problem wasn't present. D. Narrative would explore the effects of society's dominant discourse on Nate's problem; Solution Focused would determine the effect the problem was having on the family and provide psychoeducation on grief in children.

B. Narrative would separate Nate from his bad behavior and explore the effects of the bad behavior at school and home; Solution Focused would ask about times when Nate had behaved well and how the family would know the problem was gone.

Allison is a 35-year-old, woman who is referred by her EAP. She states that she's been having a hard time since she changed departments because she doesn't get along with her new boss. She says angrily, "She's always checking on me. It drives me crazy. I'm not going to put up with her attitude for much longer." When the therapist asks why the client was referred she replies, "Because she caught me drinking in the bathroom during lunch." What ethical responsibilities does the therapist have in this case? Response: A. Review options for payment of fees; Practice within scope of competence; Obtain signed consent for treatment. B. Practice within scope of competence; Manage confidentiality in contact with workplace; Monitor countertransference throughout course of treatment. C. Practice within scope of competence; Review options for payment of fees; Manage countertransference throughout course of treatment. D. Discuss risks and benefits of treatment; Limit services to permissible scope of practice; Manage confidentiality in contact with workplace.

B. Practice within scope of competence; Manage confidentiality in contact with workplace; Monitor countertransference throughout course of treatment. Rationale: Since the question is about ethical responsibilities, we are going to rule out the answers that contain legal responsibilities. Thus, answers (A), (C), and (D) are out because reviewing options for payment of fees, obtaining signed consent for treatment, and acting within scope of practice are all legal responsibilities.

A Structural Family therapist is in the middle phase of treatment with a family that has been in therapy for three months. The family sought therapy after the 36-year-old daughter and her 14-year-old son moved back home with her 66-year-old father and 65-year-old mother. The daughter had lost her job due to issues with alcohol and moved in after completing a treatment program. At the start of the session, the mother reports that three days ago the daughter relapsed and became violent when they confronted her. What action should the therapist take first? Response: A. Explore each family members perspective of the incident and create a safety plan. B. Remind the family that the therapist is a mandated reporter and obtain more details about the violent episode. C. Recommend that the daughter seek alcohol treatment immediately and create a safety plan . D. Obtain more details about the nature of the violent episode and file an APS report if necessary.

B. Remind the family that the therapist is a mandated reporter and obtain more details about the violent episode. Rationale: The code of ethics states that therapists must inform the clients of the limits of confidentiality at the start of treatment and then remind them throughout the course of treatment when necessary. We are never going to trap the client into telling us the information that later can be used to make a report. So we are going to tell the client: "I am going to stop you right there and remind you that I am a mandated reporter." Doing so goes in accordance with the code of ethics. In this scenario, there are two elders and a minor. If the violent episode was directed at one of them, a report would be required. So the best answer is (B) remind them of reporting duties then further assess. Neither (A) nor (C) acknowledge the possibility of reporting. (D) has only APS and it could be CPS.

Bill, a 42-year-old Asian-American corporate lawyer, presents for therapy on the advice of his partner. Bill states that for the past year he has had problems concentrating, moodiness, and lack of appetite. He occasionally sleeps during the day, missing work due to tiredness. He shares that he started drinking again, and admits to having four or five drinks most nights to help him fall asleep, when it used to only take two drinks. At work, Bill has been reprimanded by his supervisor for his arguments with co-workers and his decline in job performance. Bill adds, "I don't enjoy anything. My partner can't stand me this way, and I can't stand to go on this way, either. I've even thought about moving out until things can get better." Continuing, he adds, "What would my friends and family think if they knew I came in for help?" What client factors must the therapist consider in planning Bill's initial treatment? Response: A. Bill's motivation for seeking treatment; Bill's feelings regarding his loss of control and need for help; Bill's lack of social support from friends and family. B. The couple's expectations for Bill's treatment outcome; The severity of Bill's current alcohol use; Bill's feelings of shame regarding seeking help. C. Bill's motivation for seeking treatment; Bill's feelings of guilt about falling behind at work; The couple's expectations for Bill's treatment outcome. D. Psychological stressors underlying Bill's substance use; Bill's poor impulse control as demonstrated by his conflicts at work; Bill's lack of social support from friends and family.

B. The couple's expectations for Bill's treatment outcome; The severity of Bill's current alcohol use; Bill's feelings of shame regarding seeking help.

A 15-year-old, who has a diagnosis of childhood diabetes, is brought in to therapy by her parents who were sent by her MD. Recently, she has been fighting with her siblings and parents. Although she continues to receive good grades, she demonstrates an increasing need to be in control at home and is rebelling against family rules and her required diet. Which of the following conclusions should the therapist make based on this assessment information? Response: A. The symptoms are a result of mood fluctuations caused by possible misuse of prescribed medication. B. The symptoms are a result normal adolescent behavior. C. The symptoms are a result of an irregular diet causing fluctuations in her mood and behavior. D. The symptoms are a result of her need to control whatever parts of her life she can.

B. The symptoms are a result normal adolescent behavior. Rationale: The child's behaviors may represent a common response in children who feel they are lacking control in certain aspects of his/her life (D); however because they are recent, the behaviors are more likely representative of normal adolescent behavior (B). There is no indication that she is misusing her prescribed medication (A). There is a possibility that her diet is causing fluctuations in her mood (C), but the behaviors can be better explained by adolescence.

Jim, age 31, and Tina, age 32, seek therapy for intimacy issues. Tina tearfully tells of Jim's lack of affection over the past year. He can't control the amount of time he spends on the Internet at work or at home. Tina says angrily, "I can't keep living like this! He is spending all our money on porn sites. You can fix him right? What will our friends and family think if they knew that we have to come to therapy?" Tim looks out the window as she speaks. What clinical issues should the therapist address with the couple in this initial session? Response: A. The costs and benefits of internet pornography and when the issue started. B. Their expectations of therapy and their feelings about seeking help. C. Their reasons for seeking help at this time and the limits of confidentiality. D. Their values and beliefs about intimacy in their relationship and how they've coped with this conflict in the past.

B. Their expectations of therapy and their feelings about seeking help.

An adult client with developmental disabilities confides to the therapist that the receptionist at the community center often treats her like a child and she doesn't know what to do. To help the client deal with this, the therapist should: Response: A. document instances of inappropriate behavior. B. role play appropriate responses with the client. C. confront the receptionist privately. D. help the client process this and any other instance when the client was mistreated.

B. role play appropriate responses with the client. Rationale: The client clearly states that she does not know what to do. The situation described in the stem provides the therapist with an opportunity to help the client develop important social skills and role playing would achieve this task (B). Documenting inappropriate behavior (A) or confronting the receptionist privately (C) would not directly help the client. Helping the client process this and other instances of mistreatment (D) does not give the client tools to deal with this situation.

Karen, a slightly overweight 13-year-old girl, is referred for therapy by her teacher. Parents, Brad and Nina, attend the first session with Karen. Nina shares that her daughter has complained about going to school since they moved to town a month ago. Brad adds that she used to be excited to hang out with the family and now she just wants to be alone in her room. He adds, "You can fix her, right?" Karen glares at her father and says, "Fix me? You're the ones who needs help". How should the therapist clinically manage the ethical responsibilities in this case? Response: A. Address clients' expectations for treatment; Discuss risks and benefits of being in therapy; Set fees prior to the first session. B. Manage the fee; Determine who the client is and the nature of the therapist's relationship with the parties involved in treatment; Provide referrals if family decides not to continue therapy C. Address clients' expectations for treatment; Inform clients of the limits of confidentiality; Determine who the client is and the nature of the therapist's relationship with the parties involved in treatment. D. Address clients' expectations for treatment; Manage the fee; Obtain a signed release from each of them prior to speaking with Karen's teacher.

C. Address clients' expectations for treatment; Inform clients of the limits of confidentiality; Determine who the client is and the nature of the therapist's relationship with the parties involved in treatment.

A woman who has been in therapy for a couple of years finds herself at a crossroads in her life. She feels unhappy in her current career and relationship. She tells you, "I feel confused and I'm having a difficult time making sense of things... I think I want to sell all my possessions and go live on a boat and travel the world." As a Depth-Oriented psychotherapist what would you do next? Response: A. Engage the client by asking her why she has discontent in her current situation and explore the lack of meaning in her life. B. Ask the client what she feels her life purpose is and if she is fulfilling that purpose in her life. C. After acknowledging her feelings of unhappiness and confusion, invite the client to imagine her life on the boat and ask her what feelings or thoughts arise as she thinks about this life that she desires. D. After acknowledging her feelings of unhappiness and confusion, explore her various options to help her make decision about her life's purpose.

C. After acknowledging her feelings of unhappiness and confusion, invite the client to imagine her life on the boat and ask her what feelings or thoughts arise as she thinks about this life that she desires. Rationale: (C) is the correct answer. As a depth psychotherapist, you would want to engage with the image that the client has brought to the session of living on the boat. Exploration of this image would deepen the client's awareness of how she is feeling. (A) is a good option but it doesn't engage the image. (B) may come later but it is not meeting the client where she is in the moment. (D) is a more directive approach.

Melissa is a 44 year-old single mother of three children, Kristin age 17, Julie age 14, and Brad age 12. Melissa reports that Kristin is highly disruptive at home and at school and she suspects that she is using drugs and alcohol. She notes that Julie and Brad have become more withdrawn in response to Kristin's behavior and they spend most of their time at home in their rooms. How would a Strategic Family therapist intervene in the middle phase of treatment? Response: A. Request that the children sit together in the session and ask Melissa and Kristin to take turns starting their fights. B. Identify times when Kristin wasn't disruptive and encourage the family not to change. C. Ask Kristin to be extra disruptive every other night and ask Melissa and Kristin to take turns starting their fights. D. Ask each family member to describe what they want from therapy and encourage the family not to change.

C. Ask Kristin to be extra disruptive every other night and ask Melissa and Kristin to take turns starting their fights.

A 24-year-old Asian-American woman is referred to a therapist by her doctor. She tearfully reports that six months ago she was diagnosed with Lupus. She shares that her boyfriend of two years broke up with her a few months ago because he couldn't deal with the news and that she has been having a hard time getting out of bed to get to work. She recently moved in with her parents and states, "My parents say that I am depressed but my doctor says it is part of the disease." She appears clean but disheveled. What action should the therapist take first: Response: A. Provide the client with psychoeducation about how Lupus causes depression. B. Obtain a signed release of information to consult with the referring doctor. C. Ask how the client views the impact of Lupus on her mental health. D. Explore how Lupus is viewed in Asian-American culture.

C. Ask how the client views the impact of Lupus on her mental health. Rationale: The client has indicated that her parents and her doctor have different views of her symptoms. The best place to start would be asking her views on the problem (C). The client has had a lot of changes in the past few months that are likely to affect her mental health in addition to the Lupus. (A) sides with the doctor without taking in to account the other issues. (B) could be done as some point, but not when the therapist is initially connecting with the client. (D) could also be done at some point once the therapist is cleared on how the client views her situation.

A family of three presents for treatment after being referred by a family friend. Lucas, 42, states that he has recently announced to the family that he is transgender and would like to begin his transition in the near future. His wife, Amelia, 35, tells the therapist that she has been completely numb since receiving her husband's news, unable to imagine what her life is going to be like. Their son Ethan, 14, reveals his latest involvement with oxycodone. "Once kids at school learned about my dad, the ridicule has not stopped. And my parents are so preoccupied with their own problems that nobody even notices that I am high most of the time." What initial interventions should the therapist consider in the case presented in the vignette? Response: A. Encourage couple to practice active listening; encourage Ethan to seek substance abuse treatment; explore Amelia's reaction towards Lucas' transition. B. Arrange to interview each member separately; encourage Ethan to seek substance abuse treatment; assess Amelia for symptoms of depression. C. Assess Ethan for suicidal ideation and intervene accordingly; provide Lucas with appropriate referrals to support his transition; assess Amelia for symptoms of depression. D. Assess Ethan for suicidal ideation and intervene accordingly; provide Amelia with a referral to a support group for people whose spouses are transitioning; set Lucas up with individual therapy.

C. Assess Ethan for suicidal ideation and intervene accordingly; provide Lucas with appropriate referrals to support his transition; assess Amelia for symptoms of depression.

An couple in their 70s, Elise and Jim, come to therapy due to conflict over their granddaughter Jessica, age 26. A year ago, after suffering a near-fatal car accident, Jessica, along with her unemployed husband Matt, age 25, moved in with them. Jim complains, "Jessica lays around doing nothing all day, even though she recovered from her injuries months ago. They should both be out looking for work. They can't stay with us forever!" Elise appears upset and states, "My grandbaby can stay with us as long as she needs to! I just thank God that she's alive and in one piece." Jim says he suspects his grandson-in-law is an alcoholic. He reports that he heard the couple fighting violently last night in their room but was afraid to intervene "after what happened last time." What interventions would an Object Relations therapist use in the initial treatment plan in the case described in the vignette? Response: A. Create the space in which the couple can share their anxieties regarding the current situation and serve as a transitional object for Jim and Elise as they move towards differentiation. B. Interpret the mental representations of Jessica and her husband that Jim and Elise each hold and enable the couple to act as if they are on the same page about their multigenerational household. C. Create the space in which the couple can share their anxieties regarding the current situation and interpret and explore the mental representations of Jessica and her husband that Jim and Elise each hold. D. Facilitate a dialogue between Jim and Elise about the impact of Jessica's car accident on their family and enable the couple to act as if they are on the same page about their multigenerational household

C. Create the space in which the couple can share their anxieties regarding the current situation and interpret and explore the mental representations of Jessica and her husband that Jim and Elise each hold. Rationale: An Object Relations therapist would indeed create the space in which the couple can share their anxieties regarding the current situation in the initial stage of treatment. So the first half of answer (A) is good. The second half, however, is a mumbo jumbo combination of both Bowenian and Object Relations terms. Besides, Jim and Elise have not indicted the desire to differentiate. The first half in answer (B) is a good initial stage Object Relations intervention, but the second half talks about a multigenerational household, which is a Bowenian term. Plus "acting as if" is an intervention appropriate for Adlerian therapist. Answer (C) is correct since both parts are good initial stage Object Relations interventions. Answer (D) is out due to the second half that has been discussed in (B).

A 35-year-old lawyer is referred to therapy by his physician for symptoms of anxiety and panic. His physician wants the client to initiate therapy before prescribing medication. The client, however, believes that only medication will control his symptoms and that therapy is a "waste of time." Which of the following interventions would a Cognitive Behavioral therapist use to address the client's participation in therapy? Response: A. Interpret the client's distorted cognitions and identify their connection to his current symptoms of anxiety and panic. B. Explore the client's disappointment with his physician and validate his automatic thoughts as a logical consequence. C. Examine the client's assumptions regarding treatment and collaborate with him to promote a shift in personal conclusions. D. Provide psychoeducation on the benefits of Cognitive Behavioral therapy.

C. Examine the client's assumptions regarding treatment and collaborate with him to promote a shift in personal conclusions. Rationale: The best answer is C as it attends to his views of therapy. CBT therapists don't interpret, so answer A is out. There isn't an indication of the client being "disappointed" with the doctor so that knocks out B. You could do D after to do C

During an initial meeting, a client complains of depression. The client reports a history of regular cocaine use, although he reports refraining from use for several days and a desire to maintain sobriety. The therapist's first intervention would be to: Response: A. Refer client for a psychiatric evaluation. B. Recommend the client attends NA groups in conjunction with individual therapy. C. Explain to client that depression is a common withdrawal symptom. D. Explore client's level of motivation to change and maintain sobriety.

C. Explain to client that depression is a common withdrawal symptom. Rationale: The correct answer is (C) because prior to referring the client to NA (B) or for a psychiatric evaluation (A), and even prior to continuing with assessment (D), the therapist should first educate the client on the nature of his/her symptoms. Understanding that repeated use of cocaine depletes the brain of those neurotransmitters necessary to sustain normal mood resulting in depression will help the client grasp and deal with the reality of the situation.

A therapist meets with a 30-year-old man who is having relationship problems. He reports that he's been dating a woman for almost a year and is feeling conflicted over the direction of the relationship. He found the therapist through his insurance panel and would like to use his insurance to pay for his sessions. After completing an assessment, the therapist informs the client that his presenting issues will not be covered by his insurance. The man becomes upset, tells the therapist he has to use insurance or he cannot afford therapy, and asks the therapist to help him make it work. What ethical and legal obligations does the therapist have in this case? Response: A. Discuss treatment options and encourage him to find a low or no fee clinic. B. Diagnose the client with adjustment disorder and submit claim to insurance. C. Explain why insurance cannot be used and offer him a list of low fee providers. D. Offer a sliding scale fee to client based on his ability to pay.

C. Explain why insurance cannot be used and offer him a list of low fee providers. Rationale: Since the client is insisting on using his insurance, we have to respond to the client's request. So we are going to do (C) first. (A) can be done after such explanation. (B) would constitute fraud since it is illegal to make up a diagnosis simply due to reimbursement needs. (D) can also be done, but only after the explanation regarding insurance has been given.

Maria, a Mexican American, brings her 58-year-old mother, Ana, in for therapy. Maria says that she is very concerned about her mother, because she has not been the same since her husband died four weeks ago. Maria relays that her mother hasn't been able to get out of bed. Ana reports seeing and hearing her husband everywhere. According to Maria, her mother never had any type of psychological symptoms previously. What should the therapist do? Response: A. Refer Ana to an M.D. B. Refer Ana to a psychiatrist for medication evaluation. C. Explore their cultural views about grief and dying. D. Refer Ana to a psychiatrist if symptoms persist after two months.

C. Explore their cultural views about grief and dying. Rationale: A complete assessment is the first thing that the therapist should do. Exploring family's cultural views about grief and dying (C) should be part of the assessment process. If communicating with the spirit of the dead is a normal part of the culture, then referring Ana to an M.D. (A) or a psychiatrist (B) would not be necessary. The therapist might refer Ana to an M.D. (D) if the symptoms are persistent and abnormal for the client but only after completing comprehensive assessment.

A client in an initial interview reveals feelings of anxiety, guilt, and hopelessness about the future. The initial diagnostic impression of generalized anxiety disorder is reached. The client further relates that she frequently gets angry with her boyfriend for no reason. After an initial medication evaluation, a Cognitive therapist would MOST likely focus treatment on: Response: A. Assisting the client in identifying the historical origin of feelings of anxiety. B. Teaching relaxation skills. C. Exploring her underlying thoughts and beliefs about herself and others. D. Identifying the defense mechanisms she uses to cope with her anxiety.

C. Exploring her underlying thoughts and beliefs about herself and others. Rationale: Answer (A) is wrong since a Cognitive therapist does not care about the historical origin of feelings. Answer (B) is out since relaxation skills training is more of a middle stage intervention. Answer (C) is the best answer amongst the ones provided, since a Cognitive therapist would focus on client's underlying thoughts and beliefs about herself and others, and it is consistent with an initial assessment stage of treatment. Answer (D) is an intervention appropriate for a Psychodynamic therapist and not a Cognitive one.

A 25-year-old man has been meeting with a therapist for several months. He has tears in his eyes as he reports to the therapist that his girlfriend of three years broke up with him over the weekend. He tells the therapist that he is beside himself and doesn't think he will ever have a healthy relationship. He explains that he tried to make the relationship work, but his girlfriend was always disappointed in him. He states, "I am such a loser and will be alone the rest of my life. I can't handle this anymore." How would a Cognitive Behavioral therapist intervene in the middle stage of treatment? Response: A. Identify the types of cognitive distortions the client is using and challenge the client's negative thoughts. B. Teach the client to track his thoughts and develop healthier, alternative thoughts. C. Have the client examine the evidence that supports his negative beliefs as well as the evidence that contradicts these thoughts. D. Have the client identify times in his life when he had healthy relationships and did not feel like a "loser."

C. Have the client examine the evidence that supports his negative beliefs as well as the evidence that contradicts these thoughts.

A man is referred to therapy following a noticeable decline in his job performance. During the initial assessment, the client talks about his difficult childhood, physical abuse by his parents and two divorces. He is afraid that his current girlfriend is cheating on him and attributes his recent job problems to his chaotic relationship with her. He acknowledges drinking alcohol several times a week to help him relax but does not feel it is a problem. The therapist should initially clarify: Response: A. His potential job loss B. His childhood physical abuse C. His substance use D. His problems with his relationship

C. His substance use Rationale: The potential issue of substance abuse (C) must be addressed as an underlying issue before dealing with other issues presented. Once substance abuse is addressed, the therapist can focus on the client's potential job loss (A), childhood abuse (B), and problems with his relationship (D).

A 54-year-old woman is referred by her doctor after a recent panic attack. The woman shares that she has been having issues with co-workers and is afraid she may get fired. She states that her husband is her main support, but he is a corporate lawyer who travels a lot; they rarely see each other during the week. She has two grown children who are attending university out-of-state. She has few friends and shares that it has always been hard for her to trust people. Besides the client's self-report about her issues, what additional source of data would an Object Relations therapist use to formulate a complete clinical assessment? Response: A. A report from the husband. B. A reports from the doctor. C. How the client and the therapist interact. D. A questionnaire filled out by the client.

C. How the client and the therapist interact. Rationale: Object-Relations is an offshoot of psychoanalytic theory where the relationship between a therapist and a client acts as one of the most important sources of information. Thus, (C) is the best answer, as it goes along with the main tenet of this theory. The rest of the answers are other good sources of information, but they are not as specific to Object-Relations theory as is answer (C).

A 4 year-old is brought in by her parents for help with her tantrums. The parents share that they differ in how they respond. The father states, "my wife always gives in." The mother states, "he just ignores her completely." The therapist works with them to develop a consistent, age-appropriate behavioral plan. After three days the mother calls and states that her 4-year-old's tantrums are getting worse since they have started implementing the behavioral plan. She informs the therapist that they want to stop therapy. How should the therapist respond to the mother's statement? Response: A. Discuss the potential impact of premature termination. B. Explore her concerns about the interventions. C. Inform mother that an increase in problematic behavior is common at the onset of behavioral therapy. D. Validate client's feelings and encourage the client to stick it out.

C. Inform mother that an increase in problematic behavior is common at the onset of behavioral therapy.

A therapist conducts an intake interview with a 19-year-old woman who was driven to the session by her father. At the end of the session, the client's father enters the office and demands to see the therapist's notes on the interview. What should the therapist do? Response: A. Schedule a joint meeting to discuss the records. B. Ask the father to submit his request in writing. C. Inform the father of the requirement for disclosure. D. Explain that records are the property of the agency.

C. Inform the father of the requirement for disclosure. Rationale: The therapist must protect the adult client's confidentiality and would need consent from the client prior to sharing information with the father (C). The therapist would be breaking confidentiality by scheduling a meeting with the father (A) or releasing records based on the father's written request (B). Explaining that the records are the property of the agency (D) does not directly address the request.

Rosaria, a 70-year-old woman, is referred by her doctor. For some time now, Rosaria has been experiencing trouble recalling recent events or recognizing people and places. The doctor gave her the diagnosis of dementia. Rosaria is upset and scared. She says that she wants to involve her entire family in therapy with the therapist. What should the therapist do first? Response: A. Conduct research on dementia in order to gain knowledge of the disease. B. Obtain a release to speak with her doctor and determine the stage of her disease. C. Make an initial appointment just for Rosaria. D. Make an initial appointment for Rosaria's entire family.

C. Make an initial appointment just for Rosaria. Rationale: Meeting just with the client (C) would be what the therapist should do first in order to complete assessment and come up with a treatment plan. Conducting research about the disease (A) or speaking with her doctor (B) might be part of the treatment plan at a later point. Inviting her family (D) at this point is premature. The therapist needs to find out when might be a good time to do so and what might be the best way to go about it. As it stands, she was not referred for family therapy but for individual, and the therapist very well might honor her request once the therapist completes a proper assessment.

A teacher asks a therapist working in the school for professional advice about a six-year-old student who has been diagnosed with ADHD and who is having difficulties getting along with classmates. After offering information to the teacher and discussing options with the parents, the therapist should next: Response: A. Enroll the child in an after-school social club. B. Refer the child for a medication evaluation. C. Refer the child to a social skills group. D. Recommend the parents request an aide to assist with the child in class.

C. Refer the child to a social skills group. Rationale: It is beneficial for children diagnosed with ADHD to participate in social skills groups (C), especially when it is noted that the child is having difficulties getting along with other students. An after-school social club (A) might not be appropriate until the child participates in a group that will help him build skills. The referral for a medication evaluation (B) and the need for an aide (D) might be something the therapist deems appropriate in the future, but the first step would be a referral to a social skills group.

A therapist sees a teenager with a diagnosis of bulimia nervosa and a recent pattern of cutting. The family physician believes hospitalization is necessary. A psychiatrist has diagnosed a comorbid condition of major depressive disorder and recommends intensive outpatient treatment. The client and her parents are confused and frightened. The therapist should first: Response: A. Recommend outpatient therapy with the support of a dietician. B. Discuss the importance of the client's health with the family. C. Schedule a case conference with the professionals involved. D. Help the family choose a treatment option.

C. Schedule a case conference with the professionals involved. Rationale: The client's behaviors can be considered high-risk and the most appropriate course of action should be coordinated among all the professionals working with the client (C). This would be the best course of action, since the code of ethics emphasizes the importance of collaborating with all health care providers involved in our clients' cases. This way the providers can come to a unified solution and present it to the client without confusing the client. Following the case conference, the therapist and other professionals may make recommendations (A), discuss the client's health with the family (B), and help the family to choose treatment options (D).

A 42-year-old stylish and sophisticated actor seeks therapy on the advice of his uncle. The client describes a history of falling in love with many beautiful, wise women, marrying them, but being disappointed shortly after getting into the relationship with them. Each of his 3 marriages has ended in a bitter divorce. With his eyes looking down, the client notes: "All I want is to be loved and nurtured, but none of them lives up to the image they initially present. " How would an Object Relations therapist conceptualize this case? Response: A. View his marriages as attempts to overcome the client's perceived inferiorities. B. Appreciate the client's repressed sexual instincts and wonder about his feelings of jealousy towards his father. C. See the client's inner world as filled with idealized representations of women that come into conflict with the realities of what these women are actually like. D. Hypothesize that the early relationships with his caregivers lacked empathy and nurturance and inquire about the way his parents were raised.

C. See the client's inner world as filled with idealized representations of women that come into conflict with the realities of what these women are actually like.

Natalia, a 47-year-old, and Igor, a 49-year-old, come in with their 16-year-old daughter Olga. They are referred by Olga's school. The initial assessment indicates that Olga has all the symptoms of anorexia. Natalia complains that all their attention is focused on Olga. Who should the therapist's unit of treatment be? Response: A. The parents to strengthen their relationship. B. The daughter alone. C. The entire family. D. The mother alone.

C. The entire family. Rationale: One of the best ways to determine the unit of treatment is to see who is in the room with you. Since the whole family is here right now, and one of the presenting complaints is that the family's complete focus is on the daughter, it is safe to say that the entire family should be the unit of treatment. Seeing the daughter (B), the mother (D), or the parents (A) alone will not help the family deal with the complaint that was brought up.

Elizabeth, a 19-year-old college student, comes to therapy complaining of body image issues. She is average weight, which makes her unhappy and she wishes she were skinnier. "I am constantly on a diet," she states "and once a week I feel so bad about eating even a little bit of food that I have to throw up to feel better." Elizabeth's likely diagnosis is: Response: A. Anorexia nervosa B. Bulimia nervosa C. Unspecified eating disorder D. Body dysmorphic disorder

C. Unspecified eating disorder Rationale: Since Elizabeth is of average weight, the diagnosis of anorexia nervosa is out (A). Elizabeth does not meet full criteria for bulimia nervosa (B) because for such diagnosis to be present one must engage in binging and compensatory behaviors at least twice a week for three months. Thus, the correct answer is (C). (D) is out because her symptoms are better accounted for by the diagnosis of unspecified eating disorder.

A 42-year-old female client seeks therapy at the urging of her partner. The client is late to the first session and appears disheveled. The client begins crying and reports that she is distraught about the recent loss of her 82-year-old mother. How would a Client Centered therapist proceed in this case: Response: A. Express sympathy for the client's loss and assess for danger to self. B. Use body language to communicate empathy and provide psychoeducation about grief reactions. C. Use reflective listening to explore her reaction to her mother's death. D. Relate a personal experience of loss if it is similar to the client's experience.

C. Use reflective listening to explore her reaction to her mother's death.

A pediatrician refers the parents of a 10-year-old boy to therapy shortly after the boy was diagnosed with diabetes. The parents report the family is facing financial problems, and numerous other challenges that are causing them significant stress. The mother begins crying, sharing she is so overwhelmed by the challenges they are facing and feels guilty about her son's diagnosis. Considering the various problems the family is confronting, the therapist should consider which of the following actions? Response: A. Offer to work individually with the son to address his emotional response to the diagnosis. B. Offer to work with the mother and son to address feelings of guilt and stress related to the recent diagnosis and other stressors. C. Validate the parents' emotional responses to their different stressors and help them prioritize treatment goals. D. Schedule a session to meet with the entire family to determine if family or individual therapy is necessary.

C. Validate the parents' emotional responses to their different stressors and help them prioritize treatment goals.

A non-profit agency supervisor is assigned a new supervisee. During the initial supervisory session, the supervisee is asked to describe past practice experience, perceived strengths and weaknesses, and goals for supervision. The supervisor discusses the purpose of supervision, the role of the supervisor, and the expectations for the supervisee. The supervisor also states that according to legal and ethical guidelines: Response: A. the supervisor is the one who holds responsibility. B. the supervisee is the one who holds responsibility. C. both supervisor and supervisee are held equally responsible. D. the agency's clinical director solely holds responsibility.

C. both supervisor and supervisee are held equally responsible. Rationale: (C) Both the supervisor and the supervisee hold equal responsibility. The supervisor is responsible for telling the supervisee how to handle paperwork, agency procedures, how to deal with difficult situations, and clinical matters. The supervisee, however, is also responsible for his/her actions. We all hold individual responsibility to know when to break confidentiality for instance and lack of knowledge of such guidelines does not act as an acceptable excuse. The clinical director does hold responsibility as the ultimate employer but does not solely hold responsibility (D).

A 34-year-old woman seeks treatment for increasing symptoms of anxiety. She is the single mother of a 4-year-old girl, after having separated from her daughter's father 3 months ago. She reports that their separation was mutual and they continue to be friends, but she states that her feelings of anxiety are increasing. She is increasingly concerned about her ability to be a single parent and has recently been considering sending her daughter to live with the father. She reports feeling disconnected from her daughter and unsure of how to "get things back to normal." What should the therapist do first? Response: A. Discuss a referral to a psychiatrist for medication evaluation. B. Offer a joint session with the client's ex to discuss ways to split the daughter's time between the two of them. C. Assess whether there has been any recent incidents that contributed to client's increased anxiety. D. Acknowledge client's feelings of distress and confusion.

D. Acknowledge client's feelings of distress and confusion. Rationale: This question is asking what the therapist should do first. The therapist should validate and acknowledge the client's feelings (D) prior to taking any additional steps. The therapist could choose to use any of the other interventions listed in answers (A), (B), and (C), but only after acknowledging the client's feelings.

A therapist is treating a 54-year-old man who is involved in a contentious custody battle with his ex-wife. The therapist receives a subpoena for client's records from the wife's lawyer. How should the therapist proceed? Response: A. Contact the client and discuss the pending release. B. Provide a copy of the records immediately. C. Consult with a lawyer regarding the matter. D. Assert privilege on behalf of the client.

D. Assert privilege on behalf of the client.

Jose, a 32-year-old accountant, is referred to therapy by his EAP. He is approved for 8 treatment sessions. The referral from the EAP states that Jose has poor time management skills and has recently had issues with his productivity. Jose shares that his boyfriend was recently transferred to another city for work and it has caused a strain on their relationship and his overall health. He shares that he likes his job but just can't concentrate like he used to. What would a Solution Focused therapist do in the final phase of treatment? Response: A. Process his feelings about termination and compliment his progress and growth. B. Identify his resources and strengths and identify possible setbacks and ways to stay on track. C. Identify what "productive" would look like and compliment his progress and growth. D. Compare his scale of current functioning to the initial scale and identify possible setbacks and ways to stay on track.

D. Compare his scale of current functioning to the initial scale and identify possible setbacks and ways to stay on track. Rationale: Answer (A) is a good answer, yet it is not specific to a Solution Focused theory, and since that is what the question is asking, the right answer will include elements from the theory. Answer (B) is not the best answer since identifying strengths and resources are indicative of a middle stage of this theory. Answer (C) includes an intervention ("Identify what 'productive' would look like") that would be better fitting in the beginning stage of treatment. Answer (D) is the best answer since, as a Solution Focused therapist, we would utilize scaling and compare how the client is doing now as compared to how he was doing when the treatment began. The second half of the answer is also a fitting Solution Focused final stage intervention.

A 36-year-old lawyer is referred by his couple's therapist for individual therapy. The client and his wife have been in couple's therapy for 2 months. They initiated therapy after the wife caught the husband cheating on her with his co-worker. The client reports that he has ended the affair but is uncertain whether he wants to stay married to his wife of four years. The wife is willing to make amends and is eager to start a family. The client reports that he thinks he'd be an awful father and doesn't know if he should have kids. How would a Client-Centered therapist proceed in the initial phase of treatment? Response: A. Engage in reflective listening; determine whether client wants to stay married; clarify his current situation and feelings B. Explore client's early relationships; accept the client and his past; clarify his current situation and feelings C. Engage in reflective listening; accept the client and his past; identify client's underlying beliefs about the situation D. Engage in reflective listening, accept the client and his past; clarify his current situation and feelings

D. Engage in reflective listening, accept the client and his past; clarify his current situation and feelings

Alex, a 14-year-old, is referred to therapy by his school after being suspended for fighting. He is brought in by his parents, Carol and Bob, who share that Alex was recently kicked off the soccer team for talking back to the coach. Carol starts crying when she shares that "Alex won't listen to anyone. I don't know how to handle him. He is so moody all the time, I am afraid he must be doing drugs." Alex responds mockingly, "I'm not doing drugs. You're the one with the problem, it's the two of you who are always fighting." Bob ignores both of them and says, "Alex has diabetes, but refuses to monitor it. He's been hospitalized twice in the past two years. I travel a lot and need to focus on my job because people are being laid off. Alex just needs to grow up." Based on the information provided, what initial goals should the therapist consider presenting in this case? Response: A. Reduce Alex's substance use; Increase level of social supports; Stabilize Carol's symptoms of depression. B. Reduce Bob's job-related stress; Improve Alex's health problems; Decrease Alex's interpersonal conflicts at school. C. Improve Alex's academic performance; Improve Alex's peer relationships; Decrease marital conflict between Bob and Carol. D. Increase Alex's compliance with medical treatment; Decrease Alex's interpersonal conflicts at school; Develop alternative conflict-resolution skills.

D. Increase Alex's compliance with medical treatment; Decrease Alex's interpersonal conflicts at school; Develop alternative conflict-resolution skills.

Juan, a 24-year-old Hispanic male, is court-mandated to therapy as part of his conditions of parole. He was recently released after spending three years in prison for a felony assault charge. Juan reports a history of alcohol, crack, and crystal meth use. He shares that he has heard voices in the past and that is why he attacked a lady at the mall. He is currently in a halfway house and taking psychotropic medications. He states "I can't find a job, no one wants to hire a high school dropout. It was easier when I was locked up. All my friends are still in there. I don't want to get out of bed in the morning, there's no point. If my parole officer keeps riding me, I don't know what I'll do." Angrily he adds, "I don't see how therapy is going to help." What adjunctive resources should the therapist consider utilizing initially with Juan in this case? Response: A. GED program; MD; Substance abuse treatment center. B. Vocational skills center; Depression support group; Psychiatrist. C. MD; Narcotics Anonymous; Depression support group. D. Narcotics Anonymous; Vocational skills center; Psychiatrist.

D. Narcotics Anonymous; Vocational skills center; Psychiatrist.

Immediately following a group psychotherapy session, a group member returns to the office of the therapist to privately discuss an incident that had occurred during the session. The client reports that another member had broken a group rule by being disrespectful to other group members and asks the therapist to properly deal with the matter. The best therapeutic response would be to: Response: A. Explore the client's feelings about being shown disrespect to a member. B. Confront the client about not addressing the situation immediately in group. C. Assure the client that it is acceptable to privately report such concerns. D. Suggest the client consider bringing up this incident for discussion in the next group session.

D. Suggest the client consider bringing up this incident for discussion in the next group session. Rationale: When a group member expresses concerns about group dynamics, it is always important to deal with the issue with the group as a whole (D). Answers (A) through (C) do not provide an opportunity for the group as a whole to deal with the issue.


Set pelajaran terkait

Ch . 13 - Electrolytes, Acid & Bases Prep U

View Set

Anxiety + Depression Meds; Psych Quiz 1

View Set